Weaken the Argument CR Problem

This topic has expert replies
Junior | Next Rank: 30 Posts
Posts: 10
Joined: Thu Mar 28, 2013 5:26 am
Thanked: 1 times

Weaken the Argument CR Problem

by batwaraanirudh » Fri Sep 27, 2013 1:16 am
The dean of computing must be respected by the academic staff and be competent to oversee the use of computers on campus. The only deans whom academics respect are those who hold doctoral degrees, and only someone who really knows about computers can competently oversee the use of computers on campus. Furthermore, the board of trustees has decided that the dean of computing must be selected from among this university's staff. Therefore, the dean of computing must be a professor from this university's computer science department.

Which one of the following statements, if true, would weaken the argument?

(A) There are members of this university's staff who hold doctoral degrees and who are not
professors but who really know about computers.

(B) There are members of this university's philosophy department who do not hold
doctoral degrees but who really know about computers.

(C) Computer science professors who hold doctoral degrees but who are not members of this university's staff have applied for the position of dean of computing.

(D) Several members of the board of trustees of this university do not hold doctoral degrees.

(E) Some members of the computer science department at this university are not respected by academics in other departments.

Need help to solve this question.

OA: A

Newbie | Next Rank: 10 Posts
Posts: 9
Joined: Fri Apr 20, 2012 10:44 am

by beesy » Fri Sep 27, 2013 1:42 am
(A)

User avatar
Legendary Member
Posts: 1556
Joined: Tue Aug 14, 2012 11:18 pm
Thanked: 448 times
Followed by:34 members
GMAT Score:650

by theCodeToGMAT » Fri Sep 27, 2013 3:28 am
Answer [spoiler]{A}[/spoiler]

Conclusion states a strong word "must" thus trying to limit the eligible people.

Answer choice A clearly says that there are more possible people in the eligibility criteria

Thus weakening the conclusion..
R A H U L

User avatar
Master | Next Rank: 500 Posts
Posts: 234
Joined: Tue Jul 16, 2013 9:00 am
Location: West Virginia
Thanked: 9 times

by Java_85 » Fri Sep 27, 2013 2:16 pm
IMO A, B is also close, but not right!

Master | Next Rank: 500 Posts
Posts: 391
Joined: Sat Mar 02, 2013 5:13 am
Thanked: 50 times
Followed by:4 members

by rakeshd347 » Sat Sep 28, 2013 3:50 am
batwaraanirudh wrote:The dean of computing must be respected by the academic staff and be competent to oversee the use of computers on campus. The only deans whom academics respect are those who hold doctoral degrees, and only someone who really knows about computers can competently oversee the use of computers on campus. Furthermore, the board of trustees has decided that the dean of computing must be selected from among this university's staff. Therefore, the dean of computing must be a professor from this university's computer science department.

Which one of the following statements, if true, would weaken the argument?

(A) There are members of this university's staff who hold doctoral degrees and who are not
professors but who really know about computers.

(B) There are members of this university's philosophy department who do not hold
doctoral degrees but who really know about computers.

(C) Computer science professors who hold doctoral degrees but who are not members of this university's staff have applied for the position of dean of computing.

(D) Several members of the board of trustees of this university do not hold doctoral degrees.

(E) Some members of the computer science department at this university are not respected by academics in other departments.

Need help to solve this question.

OA: A
A is the correct answer but we could have a healthy argument over B too. I doubt it if GMAC makes these sort of CR questions. Whats the source of the question.

User avatar
Legendary Member
Posts: 1556
Joined: Tue Aug 14, 2012 11:18 pm
Thanked: 448 times
Followed by:34 members
GMAT Score:650

by theCodeToGMAT » Sat Sep 28, 2013 4:13 am
rakeshd347 wrote: A is the correct answer but we could have a healthy argument over B too. I doubt it if GMAC makes these sort of CR questions. Whats the source of the question.
B cannot be correct; the Premises clearly points out two conditions:

- Doctrol Degree

- Knowledge of Computers

B answer choice is not fulfilling the criteria itself.. So, it cannot weaken the conclusion

For example: Admission in College A requires minimum GMAt score of 700 & CGP 4.. I have not given GMAT but my CGP is 5.. Does this have any impact on the eligibility.. No..

Whereas, Choice A rightly points out an exceptional case.
R A H U L